*** Official AP Physics B Discussion Thread 2013-2014 ***

<p>@Vigilante13 The purpose for saying to draw a solid line is to differentiate it from the dashed line in part ii. A parabola is a line, a line does not have to be straight.</p>

<p>Also, at point D: His velocity is 0 at that point so where is this horizontal velocity coming from?</p>

<p>@kingofxbox99‌ lol the VERTICAL components cancel but the horizontal velocity stays the same? What opposes his horizontal motion lol?</p>

<p>@Vigilante13 Are you referring to part i or part ii of that question?</p>

<p>By the way, this isn’t a centripetal question.</p>

<p>@kingofxbox99‌ I’m referring to part ii. Your “solution” said it was pointed downward. If air resitance was negligible, then how does the horizontal component of velocity suddenly vanish?</p>

<p>Do you agree that at point D the person’s velocity is 0?</p>

<p>for what reason?</p>

<p>It’s vital to answering the question. At point D, would you say the person’s velocity is 0?</p>

<p>no I would not, there is a still a force of gravity, and his velocity still remains horizontal.</p>

<p>Ok, at point A, ALL of his energy is in the form of gravitational potential energy, mgh. He has a speed of 0 at point A. At the instant he reaches point D, ALL the kinetic energy gained from the swing is converted back into gravitational potential energy, since he is at the same height he started at. According to the conservation of energy (neglecting air resistance, friction, etc) he cannot have additional kinetic energy at point D because he did not have any additional energy at point A, an equivalent height. Therefore he has no velocity at the instant he is at Point D. Because his velocity is 0 (horizontal and vertical) when he lets go, he will only accelerate vertically due to gravity. There is nothing accelerating him horizontally, so his horizontal component will remain 0 (due to Newton’s laws of motion), and will therefore fall down in a straight, vertical line. </p>

<p>@kingofxbox99, okay I see what you are saying now. Now it makes more sense. Lol sry for doubting you. would I get -2 off for that since I draw it correctly (for parts i and ii; I’m assuming that the answer given would be a parabola and not an actual line). I can’t believe I didn’t see that lol.</p>

<p>Yeah, most likely only 2 points. During the exam, you’re stressed and sometimes miss obvious things: I completely failed on the buoyant force question (I randomly forgot that rho referred to the water, I said it was the block’s density) :P</p>

<p>haha lol man I feel you. I got like 60-65 right on the mc though, so even if I fail the frqs, I got the high mc score to keep me at a 5. think you got a 5?</p>

<p>I think I did really good on the FRQ, but I had to guess on like 15-20 on the multiple choice. I think that I did good enough to get at least a 65 average, so I’m relatively confident in a 5. Now we just have to wait two months :(</p>

<p>I got like the first 4 frqs almost perfect (Q1: 13’/15, Q2: 10/10, Q3: 10/10, Q4: 8/10) and then Q5, the spring and the magnetic field I like completely failed (5/15 most likely) and then on 6 and 7 I didn’t get some of the parts so I just wrote formulas down, so I probably got like 5/10s on those. Surprisingly though, if you get like 0s on 3 frqs, you can still get a 5. 67% overall isn’t that high to achieve.</p>

<p>how many points do you guys think the last part of question 1 would be?</p>

<p>@kingofxbox99‌ can you answer this question for me? you can answer it here if you want: <a href=“homework and exercises - Why doesn't Lenz's law predict the behavior of a rod on springs in a magnetic field? - Physics Stack Exchange”>http://physics.stackexchange.com/questions/112761/ap-physics-b-electrcity-magnetism-frq&lt;/a&gt;&lt;/p&gt;

<p>It is Q (I’m pretty sure), because you are not pulling the rod down which causes an induced current, the current is causing the rod to go down. Because there is no induced current we can’t use Lenz’s law.</p>

<p>Using our right hand, we know our fingers point into the page, and our palms have to point down the page (since the rod was pulled down). Our fingers are pointing to the left. This means current is flowing from Q to P, and the definition of positive current is that it flows from positive to negative terminals. Therefore, Q is the positive terminal.</p>

<p>Does that make sense? </p>

<p>When will they post the scoring guidelines?</p>

<p>@kingofxbox99‌ yeah it makes sense. I don’t understand where I got lenz’s law from. There is an induced current, but the current we are interested in comes from the battery; the induced current isn’t generated from the battery. Therefore, Lenz’s law wouldn’t work here.</p>